Linear algebra: Finding a basis for a space of polynomials

Click For Summary
The discussion centers on finding a basis for a space of polynomials defined by two subspaces, L and M. L is spanned by the polynomials {1 + t - t^3, 1 + t + t^2, 1 - t}, while M is spanned by {t^3 + t, 2 - t^3, 2 + t^3}. Participants explore the method of using RREF of an augmented matrix to derive a basis, but seek clarification on applying this to polynomial spaces. The conversation includes specific forms of vectors in L and M and their combinations in L + M and L ∩ M. The thread highlights the complexity of the problem and the need for simplification in the relationships between the coefficients of the polynomials.
gruba
Messages
203
Reaction score
1

Homework Statement


Let
a39c3e275c2591d05b49e54e3284b4ea.png
and
1179c4f0625914c5a491a155d5528415.png
are two basis of subspaces
d20caec3b48a1eef164cb4ca81ba2587.png
and [PLAIN]http://www.sosmath.com/CBB/latexrender/pictures/69691c7bdcc3ce6d5d8a1361f22d04ac.png. Find one basis of http://www.sosmath.com/CBB/latexrender/pictures/38d4e8e4669e784ae19bf38762e06045.png and [PLAIN]http://www.sosmath.com/CBB/latexrender/pictures/fd36d76c568c236aaaad68e084eef495.png.

Homework Equations


-Vector space
-Basis
-Polynomials

The Attempt at a Solution


[/B]
Could someone explain the method for finding a basis for a space of polynomials.
I know that with
2369a2488f59aa39a3fca53e0eff9f88.png
we need to find RREF of an augmented matrix,
and read a basis from matrix, but how to do it with polynomials?
 
Last edited by a moderator:
Physics news on Phys.org
Do you understand what L and M are? Can you describe these as vector spaces?
 
The first post says exactly what L and M are- it gives their bases. L is the space of polynomials spanned by \{1+ t- t^3, 1+ t+ t^2, 1- t\} so any vector in L is of the form a(1+ t- t^3)+ b(1+ t+ t^2)+ c(1- t)= -at^3+ bt^2+ (a+ b- c)t+ (a+ b+ c) for any number a, b, and c. M is the space of polynomials spanned by \{t^3+ t, 2- t^3, 2+ t^3\} so any vector is M is of the form p(t^3+ t)+ q(2- t^3)+ r(2+ t^3)= (p- q+ r)t^3+ pt+ (2q+ 2r) for any numbers p, q, and r. Any vector in L+ M is of the form (-a+ p- q+ r)t^3+ bt^2+ (a+ b- c+ p)t+ (a+ b+ c+ 2q+ 2r). Any vector in L\cap M can be written as either of those 2 first forms with -a= p+ q+ r, b= 0, a+ b- c= p, and a+ b+ c= 2q+ 2r. Simplify those.
 
  • Like
Likes gruba and blue_leaf77
HallsofIvy said:
-a= p+ q+ r
-a= p - q+ r?
 
Looks like this has turned into Halls of Ivy's homework!
 
Oh, dear!
 
Question: A clock's minute hand has length 4 and its hour hand has length 3. What is the distance between the tips at the moment when it is increasing most rapidly?(Putnam Exam Question) Answer: Making assumption that both the hands moves at constant angular velocities, the answer is ## \sqrt{7} .## But don't you think this assumption is somewhat doubtful and wrong?

Similar threads

  • · Replies 10 ·
Replies
10
Views
2K
  • · Replies 10 ·
Replies
10
Views
3K
  • · Replies 1 ·
Replies
1
Views
2K
  • · Replies 18 ·
Replies
18
Views
2K
  • · Replies 15 ·
Replies
15
Views
3K
  • · Replies 4 ·
Replies
4
Views
2K
  • · Replies 10 ·
Replies
10
Views
2K
Replies
4
Views
2K
  • · Replies 2 ·
Replies
2
Views
2K
  • · Replies 8 ·
Replies
8
Views
2K